1answer.
Ask question
Login Signup
Ask question
All categories
  • English
  • Mathematics
  • Social Studies
  • Business
  • History
  • Health
  • Geography
  • Biology
  • Physics
  • Chemistry
  • Computers and Technology
  • Arts
  • World Languages
  • Spanish
  • French
  • German
  • Advanced Placement (AP)
  • SAT
  • Medicine
  • Law
  • Engineering
erik [133]
3 years ago
6

PLEASE HELP! I WILL GIVE YOU BRAINLIEST!

Mathematics
1 answer:
VMariaS [17]3 years ago
4 0

Answer:

sry, i am doing this for points

You might be interested in
For any linear function f(x) = mx + b, when does kf(x) = f(kx)?
iogann1982 [59]

Only when <em>b</em> = 0 or <em>k</em> = 1<em>.</em>

If <em>f(x)</em> = <em>mx</em> + <em>b</em>, then

<em>f</em> (<em>kx</em>) = <em>m</em> (<em>kx</em>) + <em>b</em> = <em>mkx</em> + <em>b</em>

while

<em>k</em> <em>f(x)</em> = <em>k</em> (<em>mx</em> + <em>b</em>) = <em>kmx</em> + <em>kb</em>

The two expression are identical only if

<em>kb</em> = <em>b</em>   ===>   <em>kb</em> - <em>b</em> = 0   ===>   <em>b</em> (<em>k</em> - 1) = 0   ===>   <em>b</em> = 0 or <em>k</em> = 1

4 0
3 years ago
20. Carmen can buy bottles of paint for $2.00 each and boxes of colored pencils for $3.50 each. She can spend no more than
EleoNora [17]

Answer:

(a) The equality that express many bottles of paint, x, and boxes of colored pencils, y, Carmen can buy is  2 x + 3.5 y = 42

b)Three different solutions:

x  = 14, y = 4 is First Solution.

x  = 7, y = 8 is Second Solution.

x  = 21, y = 0 is Third Solution.

Step-by-step explanation:

Here, the cost of 1 bottle of paint = $2.00 each

The cost of 1 box of colored pencils  = $3.5 each

Let us assume the number of bottle of paints purchased = x

So, the cost of x bottle of paints   = x ( Cost of 1 bottle of paint)

= x ($2.00)  = 2 x

Also, assume the number of box of colored pencils purchased = y

So, the cost of y box of colored pencils = y ( Cost of 1 box)

= y ($3.50)  = 3.5 y

Also, the total amount to be spent on art supplies  = $42

So, the total amount spent on x paint bottles + y box of colored pencil

= $42

or,  2 x + 3.5 y = 42

a ) So, the equality that express many bottles of paint, x, and boxes of colored pencils, y, Carmen can buy is  2 x + 3.5 y = 42

b)Three different solutions:

When y = 4 ,  equation is:   2 x + 3.5(4)  = 42

or,  2 x = 42 - 14  = 28, or x = 28/2  =  14

So, x  = 14, y = 4 is First Solution.

When y = 8 ,  equation is:   2 x + 3.5(8)  = 42

or,  2 x = 42 - 28  = 14, or x = 14/2  =  7

So, x  = 7, y = 8 is Second Solution.

When y = 0 ,  equation is:   2 x + 3.5(0)  = 42

or,  2 x = 42 , or x = 42/2  = 21

So, x  = 21, y = 0 is Third Solution.

7 0
3 years ago
Given the function k(t) = 6t − 9,<br> complete the following Solve k(t) = 21.
ELEN [110]

Answer: 5(k)

Step-by-step explanation: “k”times “t” equals 6t-9 so if “k” times “t” equals 21, 6t-9=21, add 9 to 21 which equals 30 so then you divide 30 by 6 to get 5. “t”=5 which means the solution is 5(k)

(I’m just a sixth grader dont blame me if I couldn’t do this)

7 0
3 years ago
The area of a parallelogram on the map is 5cm*2. What is the area of the actual parallelogram? Explain why you can find the area
tatiyna

Step-by-step explanation:

Area=5×50m

=250m²/25000cm²

since they gave us the area of a parallelogram on a map and a scale there's no need to know the dimensions.

I'm not sure but I hope it helps

6 0
2 years ago
If AC=5cm, BC=12cm, and m AC= 40 degrees what is the radius of the circumscribed circle
melamori03 [73]
Let's assume they meant C=40 degrees.  With an angle like that they're asking for approximation; we'll oblige.

The circumradius is the product of the triangle sides divided by four times the area.

Here we have remaining side given by the Law of Cosines.

AB^2 = AC^2 + BC^2 - 2\ AC \ BC \cos C

AB^2 = AC^2 + BC^2  - 2 AC \ AB \cos C = 5^2 + 12^2 - 2(5)(12) \cos 40^\circ

AB = \sqrt{  169 - 120 \cos 40 ^\circ}  \approx 8.77921789

The area is \frac 1 2\ AC \ BC \sin C = \frac 1 2 (5)(12) \sin 40^\circ \approx 19.283628



The circumradius is  r \approx \dfrac{(5)(12)(8.77921789 )}{ 4 (19.283628) }  = 6.829019329




5 0
3 years ago
Other questions:
  • P- please<br><br> E- end<br><br> M- my<br><br> D- depression<br><br> A- and<br><br> S- suffering
    8·1 answer
  • What is 8g-6(g+1)&lt;4(2g-9)
    11·1 answer
  • Multiply: (-3/10)(-2/9)
    12·2 answers
  • Which algebraic rule describes the 180° counter-clockwise rotation about the origin?
    12·2 answers
  • Solve 12=0.9a '''''''''''''''''''''''''''''''
    11·1 answer
  • Which statement about the graphs of h(x)=−2x+4 and k(x)=4x−4 is correct?
    15·1 answer
  • If the legs of a right triangle have length 6 inches and 8 inches, what is the length of the hypotenuse in
    15·1 answer
  • The PS5 is set to release this year. It will cost $600 but the price will drop by 7.5% per year. How much will a PS5 cost in 202
    13·2 answers
  • Help please <br>........​
    7·1 answer
  • Isabella tossed a coin and spun a spinner that is divided into 3 equal sections. She did this 50 times. The results are shown in
    5·1 answer
Add answer
Login
Not registered? Fast signup
Signup
Login Signup
Ask question!